LSAT and Law School Admissions Forum

Get expert LSAT preparation and law school admissions advice from PowerScore Test Preparation.

 Administrator
PowerScore Staff
  • PowerScore Staff
  • Posts: 8917
  • Joined: Feb 02, 2011
|
#38545
Complete Question Explanation
(The complete setup for this game can be found here: lsat/viewtopic.php?t=12865)

The correct answer choice is (B)

To answer this question correctly, we need to determine the days on which J must work if H works on Wednesday. Templates 1, 2, and 3 all allow for H to work on Wednesday, thus all are in play, and since this is a Must Be True question the correct answer choice must be validated by each of them.

..... In Template 1, if H works on Wednesday (second shift), then J must work the second shift on Thursday.
..... And since we need an HJ block, H must also be on Thursday, working the first shift. That leaves J in the
..... first shift on Monday.

..... In Template 2, if H works on Wednesday (first shift), then J must work the first shift on Thursday. To
..... maintain the required HJ block, H would have to pair with the Thursday J, where H takes the second shift
..... that day. That leaves J in the second shift on Monday.

Thus far we have J on Monday and Thursday in both Templates 1 and 2.

..... In Template 3, if H works on Wednesday (first shift), then the only way to create the HJ block would be to
..... have both H and J on Monday. That leaves the other J to work the first shift on Thursday. Note that the HJ
..... Monday block is not as well-established in terms of which shift each will work compared to Templates 1 and ..... ..... ..... 2—either H or J could work shift 1 or 2—but we do still know the day they work.

Once again J is working on Monday and Thursday, meaning that those are the two days J will always work when H works Wednesday, and answer choice (B) is proven correct.

Get the most out of your LSAT Prep Plus subscription.

Analyze and track your performance with our Testing and Analytics Package.